Check my solution for discontinuous function

  • Thread starter Thread starter Government$
  • Start date Start date
  • Tags Tags
    Function
Click For Summary
The function f defined by f(x) = x for rational x and f(x) = x^2 for irrational x is proven to be discontinuous at x=2 by demonstrating that for any ε>0, there exists a δ>0 such that |f(x) - 2| exceeds ε when x is irrational and close to 2. The analysis shows that for irrational numbers near 2, f(x) approaches values significantly greater than 2, confirming discontinuity. Additionally, the function is shown to be discontinuous at all rational points and irrational points, indicating that it lacks continuity at any point in R. The discussion raises questions about specific points like x=0 and x=1, but concludes that the function is not continuous at any point. Overall, the function is entirely discontinuous across its domain.
Government$
Messages
87
Reaction score
1

Homework Statement


Let f: R -> R and defined with f(x)={ x, if x \in Q or x^2 if \in R\Q}
a) Prove that function is discontinuous at x=2;
b) Find all points for R in which function is continuous;

The Attempt at a Solution


As far as i know there are infinitely many irrationals but more importantly i can find infinitely many irrational numbers around any number. If that's the case then:

a) In a i need to show that function is discontinuous, so i need to find ε>0 such that for all δ>0, and for some x, i have | x - 2| < δ and | f(x) - 2 | > ε

So if i let ε= 1/2, i can find for any δ, no matter how small, a number x that is irrational. When number is irrational f(x)=x^2. Thus if x>2 and irrational, f(x) is going to be a bit above 4 for values close to 2 and then | f(x) - 2 | approximately 2 which is greater then ε= 1/2, and this works for any δ. Thus it is discontinuous at x=2.

b) If instead of x=2 i let x=q where, q is \in Q, i can use the same proces as in a to show that it is discontinuous at any q, where q is in Q. Also if i let x be irrational i can do same thing again and show that function is not continuous at any irrational number. So this means that it is not continuous at any point .
 
Physics news on Phys.org
what about the continuity at x = 0? and x = 1?
 
Question: A clock's minute hand has length 4 and its hour hand has length 3. What is the distance between the tips at the moment when it is increasing most rapidly?(Putnam Exam Question) Answer: Making assumption that both the hands moves at constant angular velocities, the answer is ## \sqrt{7} .## But don't you think this assumption is somewhat doubtful and wrong?

Similar threads

  • · Replies 51 ·
2
Replies
51
Views
5K
  • · Replies 11 ·
Replies
11
Views
2K
Replies
4
Views
4K
Replies
7
Views
1K
  • · Replies 3 ·
Replies
3
Views
2K
  • · Replies 3 ·
Replies
3
Views
2K
Replies
4
Views
2K
  • · Replies 8 ·
Replies
8
Views
2K
  • · Replies 20 ·
Replies
20
Views
3K
  • · Replies 3 ·
Replies
3
Views
896